Show that the closure of $A$ is the intersection of all closed sets containing $A$, topology proof needed












2












$begingroup$


I want to show that given $(X, mathcal{T})$, we define $overline A = {x in X| forall U in mathcal{T}, x in U implies U cap A neq varnothing}$ (definition of closure from Munkres), then




Show that $overline A = bigcap{C subseteq X | C text{ is closed }, A
subseteq C}$




I find this really hard to tackle because some unnaturalness in that $overline A$ is specified with respect to open sets, but then it is alternatively defined as intersection of closed sets..how to juggle between open and closed?



Several other posts also doesn't help...




  1. The closure of A is the smalled closed set containing A is proved in terms of accumulation points and limit points which I do not define


  2. Proving that the closure of a subset is the intersection of the closed subsets containing it is defined wrt of metric spaces



I am stuck on both inclusions and need some help



Attempt:



$(overline A subseteq bigcap{C subseteq X | C text{ is closed }, A
subseteq C})$






  • Let $x in overline A$, then $forall U in tau, x in U implies A
    cap U neq varnothing$. We want to show that $x in bigcap{C
    subseteq X | C text{ is closed }, A subseteq C}$


  • So we know that $x$ is contained in some $U' in mathcal{T}$ that
    has non-empty intersection with $A$, $x$ not necessarily in $A$.


  • Let $C_1$ be a closed set containing $A$, then $U' cap C_1 neq
    varnothing$. Let $C_2$ be a closed set containing $A$, then $U'
    cap C_2 neq varnothing$. Assuming $C_1 subseteq C_2$, then $U'
    cap C_1 cap C_2 neq varnothing$.


  • Continue this way, $U' cap bigcaplimits_{alpha in I} C_alpha
    neq varnothing$, where $bigcaplimits_{alpha in I} C_alpha$ is
    the intersection of all closed sets containing $A$


  • We know already that $x in U'$, but from above how can we see that $x
    in bigcaplimits_{alpha in I} C_alpha$? From figure below, it
    seems that $x$ will not be in $bigcaplimits_{alpha in I}
    C_alpha$



enter image description here



$( bigcap{C subseteq X | C text{ is closed }, A
subseteq C} subseteq overline A)$




  • Let $x in bigcap{C subseteq X | C text{ is closed }, A
    subseteq C}$, we want to show that $x in overline A$. It suffices
    to show that $forall U in mathcal{T}, x in U implies U in A
    neq varnothing$.


  • Since $x in bigcap {C}$, then there exists some closed set $C'
    subseteq X$ such that $x in C'$. Let $U in mathcal{T}$ containing
    $x$, then we will show that $U cap A neq varnothing$


  • We know that $x in C' cap U$, then $x in bigcap{C} cap U$. At this point however I still don't know whether $U cap A neq varnothing$. Couldn't we have a case in figure below where $x in bigcap {C}$ and $x in U$, but $U cap A = varnothing$?



enter image description here










share|cite|improve this question











$endgroup$












  • $begingroup$
    Hint: For the second inclusion you only need to know that $overline{A}$ is closed.
    $endgroup$
    – user60589
    Jun 9 '16 at 16:53










  • $begingroup$
    For the other inclusion assume $x notin overline{A}$. I.e. you have an open $U$ containing $x$ which has trivial intersection with $A$. Keep in mind that the complement of an open set is closed.
    $endgroup$
    – user60589
    Jun 9 '16 at 16:59












  • $begingroup$
    A limit point is just a member of the closure that is not in the interior.
    $endgroup$
    – Jacob Wakem
    Jul 12 '16 at 2:50
















2












$begingroup$


I want to show that given $(X, mathcal{T})$, we define $overline A = {x in X| forall U in mathcal{T}, x in U implies U cap A neq varnothing}$ (definition of closure from Munkres), then




Show that $overline A = bigcap{C subseteq X | C text{ is closed }, A
subseteq C}$




I find this really hard to tackle because some unnaturalness in that $overline A$ is specified with respect to open sets, but then it is alternatively defined as intersection of closed sets..how to juggle between open and closed?



Several other posts also doesn't help...




  1. The closure of A is the smalled closed set containing A is proved in terms of accumulation points and limit points which I do not define


  2. Proving that the closure of a subset is the intersection of the closed subsets containing it is defined wrt of metric spaces



I am stuck on both inclusions and need some help



Attempt:



$(overline A subseteq bigcap{C subseteq X | C text{ is closed }, A
subseteq C})$






  • Let $x in overline A$, then $forall U in tau, x in U implies A
    cap U neq varnothing$. We want to show that $x in bigcap{C
    subseteq X | C text{ is closed }, A subseteq C}$


  • So we know that $x$ is contained in some $U' in mathcal{T}$ that
    has non-empty intersection with $A$, $x$ not necessarily in $A$.


  • Let $C_1$ be a closed set containing $A$, then $U' cap C_1 neq
    varnothing$. Let $C_2$ be a closed set containing $A$, then $U'
    cap C_2 neq varnothing$. Assuming $C_1 subseteq C_2$, then $U'
    cap C_1 cap C_2 neq varnothing$.


  • Continue this way, $U' cap bigcaplimits_{alpha in I} C_alpha
    neq varnothing$, where $bigcaplimits_{alpha in I} C_alpha$ is
    the intersection of all closed sets containing $A$


  • We know already that $x in U'$, but from above how can we see that $x
    in bigcaplimits_{alpha in I} C_alpha$? From figure below, it
    seems that $x$ will not be in $bigcaplimits_{alpha in I}
    C_alpha$



enter image description here



$( bigcap{C subseteq X | C text{ is closed }, A
subseteq C} subseteq overline A)$




  • Let $x in bigcap{C subseteq X | C text{ is closed }, A
    subseteq C}$, we want to show that $x in overline A$. It suffices
    to show that $forall U in mathcal{T}, x in U implies U in A
    neq varnothing$.


  • Since $x in bigcap {C}$, then there exists some closed set $C'
    subseteq X$ such that $x in C'$. Let $U in mathcal{T}$ containing
    $x$, then we will show that $U cap A neq varnothing$


  • We know that $x in C' cap U$, then $x in bigcap{C} cap U$. At this point however I still don't know whether $U cap A neq varnothing$. Couldn't we have a case in figure below where $x in bigcap {C}$ and $x in U$, but $U cap A = varnothing$?



enter image description here










share|cite|improve this question











$endgroup$












  • $begingroup$
    Hint: For the second inclusion you only need to know that $overline{A}$ is closed.
    $endgroup$
    – user60589
    Jun 9 '16 at 16:53










  • $begingroup$
    For the other inclusion assume $x notin overline{A}$. I.e. you have an open $U$ containing $x$ which has trivial intersection with $A$. Keep in mind that the complement of an open set is closed.
    $endgroup$
    – user60589
    Jun 9 '16 at 16:59












  • $begingroup$
    A limit point is just a member of the closure that is not in the interior.
    $endgroup$
    – Jacob Wakem
    Jul 12 '16 at 2:50














2












2








2


1



$begingroup$


I want to show that given $(X, mathcal{T})$, we define $overline A = {x in X| forall U in mathcal{T}, x in U implies U cap A neq varnothing}$ (definition of closure from Munkres), then




Show that $overline A = bigcap{C subseteq X | C text{ is closed }, A
subseteq C}$




I find this really hard to tackle because some unnaturalness in that $overline A$ is specified with respect to open sets, but then it is alternatively defined as intersection of closed sets..how to juggle between open and closed?



Several other posts also doesn't help...




  1. The closure of A is the smalled closed set containing A is proved in terms of accumulation points and limit points which I do not define


  2. Proving that the closure of a subset is the intersection of the closed subsets containing it is defined wrt of metric spaces



I am stuck on both inclusions and need some help



Attempt:



$(overline A subseteq bigcap{C subseteq X | C text{ is closed }, A
subseteq C})$






  • Let $x in overline A$, then $forall U in tau, x in U implies A
    cap U neq varnothing$. We want to show that $x in bigcap{C
    subseteq X | C text{ is closed }, A subseteq C}$


  • So we know that $x$ is contained in some $U' in mathcal{T}$ that
    has non-empty intersection with $A$, $x$ not necessarily in $A$.


  • Let $C_1$ be a closed set containing $A$, then $U' cap C_1 neq
    varnothing$. Let $C_2$ be a closed set containing $A$, then $U'
    cap C_2 neq varnothing$. Assuming $C_1 subseteq C_2$, then $U'
    cap C_1 cap C_2 neq varnothing$.


  • Continue this way, $U' cap bigcaplimits_{alpha in I} C_alpha
    neq varnothing$, where $bigcaplimits_{alpha in I} C_alpha$ is
    the intersection of all closed sets containing $A$


  • We know already that $x in U'$, but from above how can we see that $x
    in bigcaplimits_{alpha in I} C_alpha$? From figure below, it
    seems that $x$ will not be in $bigcaplimits_{alpha in I}
    C_alpha$



enter image description here



$( bigcap{C subseteq X | C text{ is closed }, A
subseteq C} subseteq overline A)$




  • Let $x in bigcap{C subseteq X | C text{ is closed }, A
    subseteq C}$, we want to show that $x in overline A$. It suffices
    to show that $forall U in mathcal{T}, x in U implies U in A
    neq varnothing$.


  • Since $x in bigcap {C}$, then there exists some closed set $C'
    subseteq X$ such that $x in C'$. Let $U in mathcal{T}$ containing
    $x$, then we will show that $U cap A neq varnothing$


  • We know that $x in C' cap U$, then $x in bigcap{C} cap U$. At this point however I still don't know whether $U cap A neq varnothing$. Couldn't we have a case in figure below where $x in bigcap {C}$ and $x in U$, but $U cap A = varnothing$?



enter image description here










share|cite|improve this question











$endgroup$




I want to show that given $(X, mathcal{T})$, we define $overline A = {x in X| forall U in mathcal{T}, x in U implies U cap A neq varnothing}$ (definition of closure from Munkres), then




Show that $overline A = bigcap{C subseteq X | C text{ is closed }, A
subseteq C}$




I find this really hard to tackle because some unnaturalness in that $overline A$ is specified with respect to open sets, but then it is alternatively defined as intersection of closed sets..how to juggle between open and closed?



Several other posts also doesn't help...




  1. The closure of A is the smalled closed set containing A is proved in terms of accumulation points and limit points which I do not define


  2. Proving that the closure of a subset is the intersection of the closed subsets containing it is defined wrt of metric spaces



I am stuck on both inclusions and need some help



Attempt:



$(overline A subseteq bigcap{C subseteq X | C text{ is closed }, A
subseteq C})$






  • Let $x in overline A$, then $forall U in tau, x in U implies A
    cap U neq varnothing$. We want to show that $x in bigcap{C
    subseteq X | C text{ is closed }, A subseteq C}$


  • So we know that $x$ is contained in some $U' in mathcal{T}$ that
    has non-empty intersection with $A$, $x$ not necessarily in $A$.


  • Let $C_1$ be a closed set containing $A$, then $U' cap C_1 neq
    varnothing$. Let $C_2$ be a closed set containing $A$, then $U'
    cap C_2 neq varnothing$. Assuming $C_1 subseteq C_2$, then $U'
    cap C_1 cap C_2 neq varnothing$.


  • Continue this way, $U' cap bigcaplimits_{alpha in I} C_alpha
    neq varnothing$, where $bigcaplimits_{alpha in I} C_alpha$ is
    the intersection of all closed sets containing $A$


  • We know already that $x in U'$, but from above how can we see that $x
    in bigcaplimits_{alpha in I} C_alpha$? From figure below, it
    seems that $x$ will not be in $bigcaplimits_{alpha in I}
    C_alpha$



enter image description here



$( bigcap{C subseteq X | C text{ is closed }, A
subseteq C} subseteq overline A)$




  • Let $x in bigcap{C subseteq X | C text{ is closed }, A
    subseteq C}$, we want to show that $x in overline A$. It suffices
    to show that $forall U in mathcal{T}, x in U implies U in A
    neq varnothing$.


  • Since $x in bigcap {C}$, then there exists some closed set $C'
    subseteq X$ such that $x in C'$. Let $U in mathcal{T}$ containing
    $x$, then we will show that $U cap A neq varnothing$


  • We know that $x in C' cap U$, then $x in bigcap{C} cap U$. At this point however I still don't know whether $U cap A neq varnothing$. Couldn't we have a case in figure below where $x in bigcap {C}$ and $x in U$, but $U cap A = varnothing$?



enter image description here







real-analysis general-topology proof-verification proof-writing proof-explanation






share|cite|improve this question















share|cite|improve this question













share|cite|improve this question




share|cite|improve this question








edited Apr 13 '17 at 12:21









Community

1




1










asked Jun 9 '16 at 16:00









AåkonAåkon

4,86631761




4,86631761












  • $begingroup$
    Hint: For the second inclusion you only need to know that $overline{A}$ is closed.
    $endgroup$
    – user60589
    Jun 9 '16 at 16:53










  • $begingroup$
    For the other inclusion assume $x notin overline{A}$. I.e. you have an open $U$ containing $x$ which has trivial intersection with $A$. Keep in mind that the complement of an open set is closed.
    $endgroup$
    – user60589
    Jun 9 '16 at 16:59












  • $begingroup$
    A limit point is just a member of the closure that is not in the interior.
    $endgroup$
    – Jacob Wakem
    Jul 12 '16 at 2:50


















  • $begingroup$
    Hint: For the second inclusion you only need to know that $overline{A}$ is closed.
    $endgroup$
    – user60589
    Jun 9 '16 at 16:53










  • $begingroup$
    For the other inclusion assume $x notin overline{A}$. I.e. you have an open $U$ containing $x$ which has trivial intersection with $A$. Keep in mind that the complement of an open set is closed.
    $endgroup$
    – user60589
    Jun 9 '16 at 16:59












  • $begingroup$
    A limit point is just a member of the closure that is not in the interior.
    $endgroup$
    – Jacob Wakem
    Jul 12 '16 at 2:50
















$begingroup$
Hint: For the second inclusion you only need to know that $overline{A}$ is closed.
$endgroup$
– user60589
Jun 9 '16 at 16:53




$begingroup$
Hint: For the second inclusion you only need to know that $overline{A}$ is closed.
$endgroup$
– user60589
Jun 9 '16 at 16:53












$begingroup$
For the other inclusion assume $x notin overline{A}$. I.e. you have an open $U$ containing $x$ which has trivial intersection with $A$. Keep in mind that the complement of an open set is closed.
$endgroup$
– user60589
Jun 9 '16 at 16:59






$begingroup$
For the other inclusion assume $x notin overline{A}$. I.e. you have an open $U$ containing $x$ which has trivial intersection with $A$. Keep in mind that the complement of an open set is closed.
$endgroup$
– user60589
Jun 9 '16 at 16:59














$begingroup$
A limit point is just a member of the closure that is not in the interior.
$endgroup$
– Jacob Wakem
Jul 12 '16 at 2:50




$begingroup$
A limit point is just a member of the closure that is not in the interior.
$endgroup$
– Jacob Wakem
Jul 12 '16 at 2:50










4 Answers
4






active

oldest

votes


















4












$begingroup$

You’re getting bogged down in the details of the definitions and thereby making it much harder than it really is.



For the first inclusion, start, as you did, with an arbitrary $xinoperatorname{cl}A$. Let $C$ be any closed set such that $Asubseteq C$. Suppose that $xnotin C$: then $xin Xsetminus C$, and $Xsetminus C$ is open, so $(Xsetminus C)cap Anevarnothing$. But on the other hand we know that $Asubseteq C$, so $Acap(Xsetminus C)=varnothing$. This contradiction shows that $xin C$, and since $C$ was an arbitrary closed set containing $A$, we conclude that



$$operatorname{cl}Asubseteqbigcap{Csubseteq X:Asubseteq Ctext{ and }Ctext{ is closed}};.$$



For the opposite inclusion just observe that $operatorname{cl}A$ is one of the closed sets containing $A$, so if $xinbigcap{Csubseteq X:Asubseteq Ctext{ and }Ctext{ is closed}}$, then automatically $xinoperatorname{cl}A$. It follows that



$$bigcap{Csubseteq X:Asubseteq Ctext{ and }Ctext{ is closed}}subseteqoperatorname{cl}A$$



and hence that



$$bigcap{Csubseteq X:Asubseteq Ctext{ and }Ctext{ is closed}}=operatorname{cl}A;.$$






share|cite|improve this answer









$endgroup$













  • $begingroup$
    wow perfect solution
    $endgroup$
    – Aåkon
    Jun 9 '16 at 18:14



















1












$begingroup$

Not sure if this will help:



Definitions:



$A'$ is the set of all accumulation or limit points.



$overline{A} = A cup A'$ - this is known as the closure of $A$.




$bar{A}$ is closed.




Proof - Suppose $p$ is not in $bar{A}$. Since $p$ is not in $overline{A}$, it is not in $A$ nor is it a limit point of $A$. Therefore there must be some neighborhood $N$ of $p$ that does not intersect $A$ at all.



Can $N$ contain any limit points of $A$? No. If it contained one, $a$. Then by definition of limit point $N$ must contain another point of $A$. But $N$ contains no points of $A$, so this is ridiculous. Thus $N$ must be disjoint from both $A$ and its set of limit points, so $N subseteq overline{A}^c$, as desired. Thus the compliment of the closure is open, so the closure is closed.






share|cite|improve this answer









$endgroup$













  • $begingroup$
    thanks but I am just honestly not taught what a limit point is
    $endgroup$
    – Aåkon
    Jun 9 '16 at 18:14










  • $begingroup$
    would you like me to expand upon my answer to include information on what a limit point is?
    $endgroup$
    – Wolfy
    Jun 9 '16 at 18:15










  • $begingroup$
    Oh no that's okay, I have Munkres as reference and I think I am the only person who studies topology without knowing what limit point is. I think other people can easily understand your proof. My prof do not wish for us to know what limit point is he thinks it is detriment to learning topology...
    $endgroup$
    – Aåkon
    Jun 9 '16 at 18:17










  • $begingroup$
    I see okay well best of luck
    $endgroup$
    – Wolfy
    Jun 9 '16 at 18:17










  • $begingroup$
    A limit point is just a member of the closure that is not in the interior.
    $endgroup$
    – Jacob Wakem
    Jul 12 '16 at 3:11



















0












$begingroup$

All the points far from A form an open set (because it is a union of some open set for each of its points, definitionally). Moreover it is the largest open set without elements in A (for consider a neighborhood of a close point...) . Thus its complement, the closure, is the smallest closed set containing A.






share|cite|improve this answer









$endgroup$





















    0












    $begingroup$

    Let $x$ in $bar{A}$. Note that $bar{A}$ is closed, so $x$ is in a closed set. Suppose for some closed $C$ containing $A$ that $x$ is not in $C$. That means $x$ is in $C$'s complement, which is open. But that contradicts the fact that $x$ was in a closed set, so $x in C$. Since $x$ was arbitrary in $bar{A}$, and $C$ was an arbitrary member of all $C$ containing $A$, we have proved $bar{A} subset mathscr{C}$, where the fancy "C" is the set of all closed sets containing $A$. Keep in mind that if a set exists in every one of a collection of sets, then it must also be in their intersection.



    Now, for equality we need to show $bar{A} supset bigcap{C}$. Let $x$ in $bigcap C$. Suppose $x$ not in $bar{A}$, then $x$ is in $bar{A}^c$ which is open. But we just said $x$ is in an intersection of closed sets, so it must be in a closed set. Contradiction again.





    The one gap in this proof is the fact that these sets could have been $mathbb{R}^d$. But in that case equality is automatic.






    share|cite|improve this answer









    $endgroup$














      Your Answer





      StackExchange.ifUsing("editor", function () {
      return StackExchange.using("mathjaxEditing", function () {
      StackExchange.MarkdownEditor.creationCallbacks.add(function (editor, postfix) {
      StackExchange.mathjaxEditing.prepareWmdForMathJax(editor, postfix, [["$", "$"], ["\\(","\\)"]]);
      });
      });
      }, "mathjax-editing");

      StackExchange.ready(function() {
      var channelOptions = {
      tags: "".split(" "),
      id: "69"
      };
      initTagRenderer("".split(" "), "".split(" "), channelOptions);

      StackExchange.using("externalEditor", function() {
      // Have to fire editor after snippets, if snippets enabled
      if (StackExchange.settings.snippets.snippetsEnabled) {
      StackExchange.using("snippets", function() {
      createEditor();
      });
      }
      else {
      createEditor();
      }
      });

      function createEditor() {
      StackExchange.prepareEditor({
      heartbeatType: 'answer',
      autoActivateHeartbeat: false,
      convertImagesToLinks: true,
      noModals: true,
      showLowRepImageUploadWarning: true,
      reputationToPostImages: 10,
      bindNavPrevention: true,
      postfix: "",
      imageUploader: {
      brandingHtml: "Powered by u003ca class="icon-imgur-white" href="https://imgur.com/"u003eu003c/au003e",
      contentPolicyHtml: "User contributions licensed under u003ca href="https://creativecommons.org/licenses/by-sa/3.0/"u003ecc by-sa 3.0 with attribution requiredu003c/au003e u003ca href="https://stackoverflow.com/legal/content-policy"u003e(content policy)u003c/au003e",
      allowUrls: true
      },
      noCode: true, onDemand: true,
      discardSelector: ".discard-answer"
      ,immediatelyShowMarkdownHelp:true
      });


      }
      });














      draft saved

      draft discarded


















      StackExchange.ready(
      function () {
      StackExchange.openid.initPostLogin('.new-post-login', 'https%3a%2f%2fmath.stackexchange.com%2fquestions%2f1819839%2fshow-that-the-closure-of-a-is-the-intersection-of-all-closed-sets-containing%23new-answer', 'question_page');
      }
      );

      Post as a guest















      Required, but never shown

























      4 Answers
      4






      active

      oldest

      votes








      4 Answers
      4






      active

      oldest

      votes









      active

      oldest

      votes






      active

      oldest

      votes









      4












      $begingroup$

      You’re getting bogged down in the details of the definitions and thereby making it much harder than it really is.



      For the first inclusion, start, as you did, with an arbitrary $xinoperatorname{cl}A$. Let $C$ be any closed set such that $Asubseteq C$. Suppose that $xnotin C$: then $xin Xsetminus C$, and $Xsetminus C$ is open, so $(Xsetminus C)cap Anevarnothing$. But on the other hand we know that $Asubseteq C$, so $Acap(Xsetminus C)=varnothing$. This contradiction shows that $xin C$, and since $C$ was an arbitrary closed set containing $A$, we conclude that



      $$operatorname{cl}Asubseteqbigcap{Csubseteq X:Asubseteq Ctext{ and }Ctext{ is closed}};.$$



      For the opposite inclusion just observe that $operatorname{cl}A$ is one of the closed sets containing $A$, so if $xinbigcap{Csubseteq X:Asubseteq Ctext{ and }Ctext{ is closed}}$, then automatically $xinoperatorname{cl}A$. It follows that



      $$bigcap{Csubseteq X:Asubseteq Ctext{ and }Ctext{ is closed}}subseteqoperatorname{cl}A$$



      and hence that



      $$bigcap{Csubseteq X:Asubseteq Ctext{ and }Ctext{ is closed}}=operatorname{cl}A;.$$






      share|cite|improve this answer









      $endgroup$













      • $begingroup$
        wow perfect solution
        $endgroup$
        – Aåkon
        Jun 9 '16 at 18:14
















      4












      $begingroup$

      You’re getting bogged down in the details of the definitions and thereby making it much harder than it really is.



      For the first inclusion, start, as you did, with an arbitrary $xinoperatorname{cl}A$. Let $C$ be any closed set such that $Asubseteq C$. Suppose that $xnotin C$: then $xin Xsetminus C$, and $Xsetminus C$ is open, so $(Xsetminus C)cap Anevarnothing$. But on the other hand we know that $Asubseteq C$, so $Acap(Xsetminus C)=varnothing$. This contradiction shows that $xin C$, and since $C$ was an arbitrary closed set containing $A$, we conclude that



      $$operatorname{cl}Asubseteqbigcap{Csubseteq X:Asubseteq Ctext{ and }Ctext{ is closed}};.$$



      For the opposite inclusion just observe that $operatorname{cl}A$ is one of the closed sets containing $A$, so if $xinbigcap{Csubseteq X:Asubseteq Ctext{ and }Ctext{ is closed}}$, then automatically $xinoperatorname{cl}A$. It follows that



      $$bigcap{Csubseteq X:Asubseteq Ctext{ and }Ctext{ is closed}}subseteqoperatorname{cl}A$$



      and hence that



      $$bigcap{Csubseteq X:Asubseteq Ctext{ and }Ctext{ is closed}}=operatorname{cl}A;.$$






      share|cite|improve this answer









      $endgroup$













      • $begingroup$
        wow perfect solution
        $endgroup$
        – Aåkon
        Jun 9 '16 at 18:14














      4












      4








      4





      $begingroup$

      You’re getting bogged down in the details of the definitions and thereby making it much harder than it really is.



      For the first inclusion, start, as you did, with an arbitrary $xinoperatorname{cl}A$. Let $C$ be any closed set such that $Asubseteq C$. Suppose that $xnotin C$: then $xin Xsetminus C$, and $Xsetminus C$ is open, so $(Xsetminus C)cap Anevarnothing$. But on the other hand we know that $Asubseteq C$, so $Acap(Xsetminus C)=varnothing$. This contradiction shows that $xin C$, and since $C$ was an arbitrary closed set containing $A$, we conclude that



      $$operatorname{cl}Asubseteqbigcap{Csubseteq X:Asubseteq Ctext{ and }Ctext{ is closed}};.$$



      For the opposite inclusion just observe that $operatorname{cl}A$ is one of the closed sets containing $A$, so if $xinbigcap{Csubseteq X:Asubseteq Ctext{ and }Ctext{ is closed}}$, then automatically $xinoperatorname{cl}A$. It follows that



      $$bigcap{Csubseteq X:Asubseteq Ctext{ and }Ctext{ is closed}}subseteqoperatorname{cl}A$$



      and hence that



      $$bigcap{Csubseteq X:Asubseteq Ctext{ and }Ctext{ is closed}}=operatorname{cl}A;.$$






      share|cite|improve this answer









      $endgroup$



      You’re getting bogged down in the details of the definitions and thereby making it much harder than it really is.



      For the first inclusion, start, as you did, with an arbitrary $xinoperatorname{cl}A$. Let $C$ be any closed set such that $Asubseteq C$. Suppose that $xnotin C$: then $xin Xsetminus C$, and $Xsetminus C$ is open, so $(Xsetminus C)cap Anevarnothing$. But on the other hand we know that $Asubseteq C$, so $Acap(Xsetminus C)=varnothing$. This contradiction shows that $xin C$, and since $C$ was an arbitrary closed set containing $A$, we conclude that



      $$operatorname{cl}Asubseteqbigcap{Csubseteq X:Asubseteq Ctext{ and }Ctext{ is closed}};.$$



      For the opposite inclusion just observe that $operatorname{cl}A$ is one of the closed sets containing $A$, so if $xinbigcap{Csubseteq X:Asubseteq Ctext{ and }Ctext{ is closed}}$, then automatically $xinoperatorname{cl}A$. It follows that



      $$bigcap{Csubseteq X:Asubseteq Ctext{ and }Ctext{ is closed}}subseteqoperatorname{cl}A$$



      and hence that



      $$bigcap{Csubseteq X:Asubseteq Ctext{ and }Ctext{ is closed}}=operatorname{cl}A;.$$







      share|cite|improve this answer












      share|cite|improve this answer



      share|cite|improve this answer










      answered Jun 9 '16 at 17:40









      Brian M. ScottBrian M. Scott

      460k40516917




      460k40516917












      • $begingroup$
        wow perfect solution
        $endgroup$
        – Aåkon
        Jun 9 '16 at 18:14


















      • $begingroup$
        wow perfect solution
        $endgroup$
        – Aåkon
        Jun 9 '16 at 18:14
















      $begingroup$
      wow perfect solution
      $endgroup$
      – Aåkon
      Jun 9 '16 at 18:14




      $begingroup$
      wow perfect solution
      $endgroup$
      – Aåkon
      Jun 9 '16 at 18:14











      1












      $begingroup$

      Not sure if this will help:



      Definitions:



      $A'$ is the set of all accumulation or limit points.



      $overline{A} = A cup A'$ - this is known as the closure of $A$.




      $bar{A}$ is closed.




      Proof - Suppose $p$ is not in $bar{A}$. Since $p$ is not in $overline{A}$, it is not in $A$ nor is it a limit point of $A$. Therefore there must be some neighborhood $N$ of $p$ that does not intersect $A$ at all.



      Can $N$ contain any limit points of $A$? No. If it contained one, $a$. Then by definition of limit point $N$ must contain another point of $A$. But $N$ contains no points of $A$, so this is ridiculous. Thus $N$ must be disjoint from both $A$ and its set of limit points, so $N subseteq overline{A}^c$, as desired. Thus the compliment of the closure is open, so the closure is closed.






      share|cite|improve this answer









      $endgroup$













      • $begingroup$
        thanks but I am just honestly not taught what a limit point is
        $endgroup$
        – Aåkon
        Jun 9 '16 at 18:14










      • $begingroup$
        would you like me to expand upon my answer to include information on what a limit point is?
        $endgroup$
        – Wolfy
        Jun 9 '16 at 18:15










      • $begingroup$
        Oh no that's okay, I have Munkres as reference and I think I am the only person who studies topology without knowing what limit point is. I think other people can easily understand your proof. My prof do not wish for us to know what limit point is he thinks it is detriment to learning topology...
        $endgroup$
        – Aåkon
        Jun 9 '16 at 18:17










      • $begingroup$
        I see okay well best of luck
        $endgroup$
        – Wolfy
        Jun 9 '16 at 18:17










      • $begingroup$
        A limit point is just a member of the closure that is not in the interior.
        $endgroup$
        – Jacob Wakem
        Jul 12 '16 at 3:11
















      1












      $begingroup$

      Not sure if this will help:



      Definitions:



      $A'$ is the set of all accumulation or limit points.



      $overline{A} = A cup A'$ - this is known as the closure of $A$.




      $bar{A}$ is closed.




      Proof - Suppose $p$ is not in $bar{A}$. Since $p$ is not in $overline{A}$, it is not in $A$ nor is it a limit point of $A$. Therefore there must be some neighborhood $N$ of $p$ that does not intersect $A$ at all.



      Can $N$ contain any limit points of $A$? No. If it contained one, $a$. Then by definition of limit point $N$ must contain another point of $A$. But $N$ contains no points of $A$, so this is ridiculous. Thus $N$ must be disjoint from both $A$ and its set of limit points, so $N subseteq overline{A}^c$, as desired. Thus the compliment of the closure is open, so the closure is closed.






      share|cite|improve this answer









      $endgroup$













      • $begingroup$
        thanks but I am just honestly not taught what a limit point is
        $endgroup$
        – Aåkon
        Jun 9 '16 at 18:14










      • $begingroup$
        would you like me to expand upon my answer to include information on what a limit point is?
        $endgroup$
        – Wolfy
        Jun 9 '16 at 18:15










      • $begingroup$
        Oh no that's okay, I have Munkres as reference and I think I am the only person who studies topology without knowing what limit point is. I think other people can easily understand your proof. My prof do not wish for us to know what limit point is he thinks it is detriment to learning topology...
        $endgroup$
        – Aåkon
        Jun 9 '16 at 18:17










      • $begingroup$
        I see okay well best of luck
        $endgroup$
        – Wolfy
        Jun 9 '16 at 18:17










      • $begingroup$
        A limit point is just a member of the closure that is not in the interior.
        $endgroup$
        – Jacob Wakem
        Jul 12 '16 at 3:11














      1












      1








      1





      $begingroup$

      Not sure if this will help:



      Definitions:



      $A'$ is the set of all accumulation or limit points.



      $overline{A} = A cup A'$ - this is known as the closure of $A$.




      $bar{A}$ is closed.




      Proof - Suppose $p$ is not in $bar{A}$. Since $p$ is not in $overline{A}$, it is not in $A$ nor is it a limit point of $A$. Therefore there must be some neighborhood $N$ of $p$ that does not intersect $A$ at all.



      Can $N$ contain any limit points of $A$? No. If it contained one, $a$. Then by definition of limit point $N$ must contain another point of $A$. But $N$ contains no points of $A$, so this is ridiculous. Thus $N$ must be disjoint from both $A$ and its set of limit points, so $N subseteq overline{A}^c$, as desired. Thus the compliment of the closure is open, so the closure is closed.






      share|cite|improve this answer









      $endgroup$



      Not sure if this will help:



      Definitions:



      $A'$ is the set of all accumulation or limit points.



      $overline{A} = A cup A'$ - this is known as the closure of $A$.




      $bar{A}$ is closed.




      Proof - Suppose $p$ is not in $bar{A}$. Since $p$ is not in $overline{A}$, it is not in $A$ nor is it a limit point of $A$. Therefore there must be some neighborhood $N$ of $p$ that does not intersect $A$ at all.



      Can $N$ contain any limit points of $A$? No. If it contained one, $a$. Then by definition of limit point $N$ must contain another point of $A$. But $N$ contains no points of $A$, so this is ridiculous. Thus $N$ must be disjoint from both $A$ and its set of limit points, so $N subseteq overline{A}^c$, as desired. Thus the compliment of the closure is open, so the closure is closed.







      share|cite|improve this answer












      share|cite|improve this answer



      share|cite|improve this answer










      answered Jun 9 '16 at 17:32









      WolfyWolfy

      2,37111241




      2,37111241












      • $begingroup$
        thanks but I am just honestly not taught what a limit point is
        $endgroup$
        – Aåkon
        Jun 9 '16 at 18:14










      • $begingroup$
        would you like me to expand upon my answer to include information on what a limit point is?
        $endgroup$
        – Wolfy
        Jun 9 '16 at 18:15










      • $begingroup$
        Oh no that's okay, I have Munkres as reference and I think I am the only person who studies topology without knowing what limit point is. I think other people can easily understand your proof. My prof do not wish for us to know what limit point is he thinks it is detriment to learning topology...
        $endgroup$
        – Aåkon
        Jun 9 '16 at 18:17










      • $begingroup$
        I see okay well best of luck
        $endgroup$
        – Wolfy
        Jun 9 '16 at 18:17










      • $begingroup$
        A limit point is just a member of the closure that is not in the interior.
        $endgroup$
        – Jacob Wakem
        Jul 12 '16 at 3:11


















      • $begingroup$
        thanks but I am just honestly not taught what a limit point is
        $endgroup$
        – Aåkon
        Jun 9 '16 at 18:14










      • $begingroup$
        would you like me to expand upon my answer to include information on what a limit point is?
        $endgroup$
        – Wolfy
        Jun 9 '16 at 18:15










      • $begingroup$
        Oh no that's okay, I have Munkres as reference and I think I am the only person who studies topology without knowing what limit point is. I think other people can easily understand your proof. My prof do not wish for us to know what limit point is he thinks it is detriment to learning topology...
        $endgroup$
        – Aåkon
        Jun 9 '16 at 18:17










      • $begingroup$
        I see okay well best of luck
        $endgroup$
        – Wolfy
        Jun 9 '16 at 18:17










      • $begingroup$
        A limit point is just a member of the closure that is not in the interior.
        $endgroup$
        – Jacob Wakem
        Jul 12 '16 at 3:11
















      $begingroup$
      thanks but I am just honestly not taught what a limit point is
      $endgroup$
      – Aåkon
      Jun 9 '16 at 18:14




      $begingroup$
      thanks but I am just honestly not taught what a limit point is
      $endgroup$
      – Aåkon
      Jun 9 '16 at 18:14












      $begingroup$
      would you like me to expand upon my answer to include information on what a limit point is?
      $endgroup$
      – Wolfy
      Jun 9 '16 at 18:15




      $begingroup$
      would you like me to expand upon my answer to include information on what a limit point is?
      $endgroup$
      – Wolfy
      Jun 9 '16 at 18:15












      $begingroup$
      Oh no that's okay, I have Munkres as reference and I think I am the only person who studies topology without knowing what limit point is. I think other people can easily understand your proof. My prof do not wish for us to know what limit point is he thinks it is detriment to learning topology...
      $endgroup$
      – Aåkon
      Jun 9 '16 at 18:17




      $begingroup$
      Oh no that's okay, I have Munkres as reference and I think I am the only person who studies topology without knowing what limit point is. I think other people can easily understand your proof. My prof do not wish for us to know what limit point is he thinks it is detriment to learning topology...
      $endgroup$
      – Aåkon
      Jun 9 '16 at 18:17












      $begingroup$
      I see okay well best of luck
      $endgroup$
      – Wolfy
      Jun 9 '16 at 18:17




      $begingroup$
      I see okay well best of luck
      $endgroup$
      – Wolfy
      Jun 9 '16 at 18:17












      $begingroup$
      A limit point is just a member of the closure that is not in the interior.
      $endgroup$
      – Jacob Wakem
      Jul 12 '16 at 3:11




      $begingroup$
      A limit point is just a member of the closure that is not in the interior.
      $endgroup$
      – Jacob Wakem
      Jul 12 '16 at 3:11











      0












      $begingroup$

      All the points far from A form an open set (because it is a union of some open set for each of its points, definitionally). Moreover it is the largest open set without elements in A (for consider a neighborhood of a close point...) . Thus its complement, the closure, is the smallest closed set containing A.






      share|cite|improve this answer









      $endgroup$


















        0












        $begingroup$

        All the points far from A form an open set (because it is a union of some open set for each of its points, definitionally). Moreover it is the largest open set without elements in A (for consider a neighborhood of a close point...) . Thus its complement, the closure, is the smallest closed set containing A.






        share|cite|improve this answer









        $endgroup$
















          0












          0








          0





          $begingroup$

          All the points far from A form an open set (because it is a union of some open set for each of its points, definitionally). Moreover it is the largest open set without elements in A (for consider a neighborhood of a close point...) . Thus its complement, the closure, is the smallest closed set containing A.






          share|cite|improve this answer









          $endgroup$



          All the points far from A form an open set (because it is a union of some open set for each of its points, definitionally). Moreover it is the largest open set without elements in A (for consider a neighborhood of a close point...) . Thus its complement, the closure, is the smallest closed set containing A.







          share|cite|improve this answer












          share|cite|improve this answer



          share|cite|improve this answer










          answered Jul 12 '16 at 2:39









          Jacob WakemJacob Wakem

          1,912621




          1,912621























              0












              $begingroup$

              Let $x$ in $bar{A}$. Note that $bar{A}$ is closed, so $x$ is in a closed set. Suppose for some closed $C$ containing $A$ that $x$ is not in $C$. That means $x$ is in $C$'s complement, which is open. But that contradicts the fact that $x$ was in a closed set, so $x in C$. Since $x$ was arbitrary in $bar{A}$, and $C$ was an arbitrary member of all $C$ containing $A$, we have proved $bar{A} subset mathscr{C}$, where the fancy "C" is the set of all closed sets containing $A$. Keep in mind that if a set exists in every one of a collection of sets, then it must also be in their intersection.



              Now, for equality we need to show $bar{A} supset bigcap{C}$. Let $x$ in $bigcap C$. Suppose $x$ not in $bar{A}$, then $x$ is in $bar{A}^c$ which is open. But we just said $x$ is in an intersection of closed sets, so it must be in a closed set. Contradiction again.





              The one gap in this proof is the fact that these sets could have been $mathbb{R}^d$. But in that case equality is automatic.






              share|cite|improve this answer









              $endgroup$


















                0












                $begingroup$

                Let $x$ in $bar{A}$. Note that $bar{A}$ is closed, so $x$ is in a closed set. Suppose for some closed $C$ containing $A$ that $x$ is not in $C$. That means $x$ is in $C$'s complement, which is open. But that contradicts the fact that $x$ was in a closed set, so $x in C$. Since $x$ was arbitrary in $bar{A}$, and $C$ was an arbitrary member of all $C$ containing $A$, we have proved $bar{A} subset mathscr{C}$, where the fancy "C" is the set of all closed sets containing $A$. Keep in mind that if a set exists in every one of a collection of sets, then it must also be in their intersection.



                Now, for equality we need to show $bar{A} supset bigcap{C}$. Let $x$ in $bigcap C$. Suppose $x$ not in $bar{A}$, then $x$ is in $bar{A}^c$ which is open. But we just said $x$ is in an intersection of closed sets, so it must be in a closed set. Contradiction again.





                The one gap in this proof is the fact that these sets could have been $mathbb{R}^d$. But in that case equality is automatic.






                share|cite|improve this answer









                $endgroup$
















                  0












                  0








                  0





                  $begingroup$

                  Let $x$ in $bar{A}$. Note that $bar{A}$ is closed, so $x$ is in a closed set. Suppose for some closed $C$ containing $A$ that $x$ is not in $C$. That means $x$ is in $C$'s complement, which is open. But that contradicts the fact that $x$ was in a closed set, so $x in C$. Since $x$ was arbitrary in $bar{A}$, and $C$ was an arbitrary member of all $C$ containing $A$, we have proved $bar{A} subset mathscr{C}$, where the fancy "C" is the set of all closed sets containing $A$. Keep in mind that if a set exists in every one of a collection of sets, then it must also be in their intersection.



                  Now, for equality we need to show $bar{A} supset bigcap{C}$. Let $x$ in $bigcap C$. Suppose $x$ not in $bar{A}$, then $x$ is in $bar{A}^c$ which is open. But we just said $x$ is in an intersection of closed sets, so it must be in a closed set. Contradiction again.





                  The one gap in this proof is the fact that these sets could have been $mathbb{R}^d$. But in that case equality is automatic.






                  share|cite|improve this answer









                  $endgroup$



                  Let $x$ in $bar{A}$. Note that $bar{A}$ is closed, so $x$ is in a closed set. Suppose for some closed $C$ containing $A$ that $x$ is not in $C$. That means $x$ is in $C$'s complement, which is open. But that contradicts the fact that $x$ was in a closed set, so $x in C$. Since $x$ was arbitrary in $bar{A}$, and $C$ was an arbitrary member of all $C$ containing $A$, we have proved $bar{A} subset mathscr{C}$, where the fancy "C" is the set of all closed sets containing $A$. Keep in mind that if a set exists in every one of a collection of sets, then it must also be in their intersection.



                  Now, for equality we need to show $bar{A} supset bigcap{C}$. Let $x$ in $bigcap C$. Suppose $x$ not in $bar{A}$, then $x$ is in $bar{A}^c$ which is open. But we just said $x$ is in an intersection of closed sets, so it must be in a closed set. Contradiction again.





                  The one gap in this proof is the fact that these sets could have been $mathbb{R}^d$. But in that case equality is automatic.







                  share|cite|improve this answer












                  share|cite|improve this answer



                  share|cite|improve this answer










                  answered Dec 11 '18 at 4:12









                  ZduffZduff

                  1,7191020




                  1,7191020






























                      draft saved

                      draft discarded




















































                      Thanks for contributing an answer to Mathematics Stack Exchange!


                      • Please be sure to answer the question. Provide details and share your research!

                      But avoid



                      • Asking for help, clarification, or responding to other answers.

                      • Making statements based on opinion; back them up with references or personal experience.


                      Use MathJax to format equations. MathJax reference.


                      To learn more, see our tips on writing great answers.




                      draft saved


                      draft discarded














                      StackExchange.ready(
                      function () {
                      StackExchange.openid.initPostLogin('.new-post-login', 'https%3a%2f%2fmath.stackexchange.com%2fquestions%2f1819839%2fshow-that-the-closure-of-a-is-the-intersection-of-all-closed-sets-containing%23new-answer', 'question_page');
                      }
                      );

                      Post as a guest















                      Required, but never shown





















































                      Required, but never shown














                      Required, but never shown












                      Required, but never shown







                      Required, but never shown

































                      Required, but never shown














                      Required, but never shown












                      Required, but never shown







                      Required, but never shown







                      Popular posts from this blog

                      Biblatex bibliography style without URLs when DOI exists (in Overleaf with Zotero bibliography)

                      ComboBox Display Member on multiple fields

                      Is it possible to collect Nectar points via Trainline?